sum of reciprocal numbers of combinations











up vote
2
down vote

favorite
1












Let $^nC_k:=dfrac{n!}{k!(n-k)!}$
Please prove that,for all natural number $k≥2$, $displaystylesum_{n=k+1}^{infty}frac{1}{^nC_k}=frac{1}{k-1}$



I tried to prove by induction, but I cannot. I guess it is proved by using Tayler series for some function, but I cannot find the function.










share|cite|improve this question




























    up vote
    2
    down vote

    favorite
    1












    Let $^nC_k:=dfrac{n!}{k!(n-k)!}$
    Please prove that,for all natural number $k≥2$, $displaystylesum_{n=k+1}^{infty}frac{1}{^nC_k}=frac{1}{k-1}$



    I tried to prove by induction, but I cannot. I guess it is proved by using Tayler series for some function, but I cannot find the function.










    share|cite|improve this question


























      up vote
      2
      down vote

      favorite
      1









      up vote
      2
      down vote

      favorite
      1






      1





      Let $^nC_k:=dfrac{n!}{k!(n-k)!}$
      Please prove that,for all natural number $k≥2$, $displaystylesum_{n=k+1}^{infty}frac{1}{^nC_k}=frac{1}{k-1}$



      I tried to prove by induction, but I cannot. I guess it is proved by using Tayler series for some function, but I cannot find the function.










      share|cite|improve this question















      Let $^nC_k:=dfrac{n!}{k!(n-k)!}$
      Please prove that,for all natural number $k≥2$, $displaystylesum_{n=k+1}^{infty}frac{1}{^nC_k}=frac{1}{k-1}$



      I tried to prove by induction, but I cannot. I guess it is proved by using Tayler series for some function, but I cannot find the function.







      combinatorics taylor-expansion






      share|cite|improve this question















      share|cite|improve this question













      share|cite|improve this question




      share|cite|improve this question








      edited Nov 17 at 14:56









      Yadati Kiran

      1,245417




      1,245417










      asked Nov 17 at 14:44









      J.Rie

      183




      183






















          2 Answers
          2






          active

          oldest

          votes

















          up vote
          1
          down vote



          accepted










          That is known as the German tank problem, and is one of the fundamental Binomial Identities.






          share|cite|improve this answer




























            up vote
            0
            down vote













            This is very simple to prove. All you need to note is the following identity :
            $$
            frac 1{a(a+1)...(a+m)} = frac{1}{m}left( frac 1{a(a+1)...(a+m-1)} - frac 1{(a+1)...(a+m)}right)
            $$



            Now, consider the sum:
            $$
            sum_{n=k+1}^{k+m} frac 1{binom nk} = k! times sum_{i=1}^{i=m} frac{i!}{(k+i)!} \ = k! sum_{i=1}^{i=m}frac 1{(i+1)(i+2)...(i+k)} \ = k! sum_{i=1}^{i=m} frac{1}{k-1}left(frac{1}{(i+1)...(i+k-1)} - frac 1{(i+2)...(i+k)}right) \ = frac{k!}{k-1}sum_{i=1}^{i=m} left(frac{1}{(i+1)...(i+k-1)} - frac 1{(i+2)...(i+k)}right) \ = frac{k!}{k-1} left(frac 1{k!} - frac 1{(m+2)...(m+k)}right) \ = frac 1{k-1} - frac{k!}{(k-1)(m+2)...(m+k)}
            $$



            By letting $m$ go to infinity, the answer is clearly $frac 1{k-1}$, since the second term goes to zero.






            share|cite|improve this answer





















              Your Answer





              StackExchange.ifUsing("editor", function () {
              return StackExchange.using("mathjaxEditing", function () {
              StackExchange.MarkdownEditor.creationCallbacks.add(function (editor, postfix) {
              StackExchange.mathjaxEditing.prepareWmdForMathJax(editor, postfix, [["$", "$"], ["\\(","\\)"]]);
              });
              });
              }, "mathjax-editing");

              StackExchange.ready(function() {
              var channelOptions = {
              tags: "".split(" "),
              id: "69"
              };
              initTagRenderer("".split(" "), "".split(" "), channelOptions);

              StackExchange.using("externalEditor", function() {
              // Have to fire editor after snippets, if snippets enabled
              if (StackExchange.settings.snippets.snippetsEnabled) {
              StackExchange.using("snippets", function() {
              createEditor();
              });
              }
              else {
              createEditor();
              }
              });

              function createEditor() {
              StackExchange.prepareEditor({
              heartbeatType: 'answer',
              convertImagesToLinks: true,
              noModals: true,
              showLowRepImageUploadWarning: true,
              reputationToPostImages: 10,
              bindNavPrevention: true,
              postfix: "",
              imageUploader: {
              brandingHtml: "Powered by u003ca class="icon-imgur-white" href="https://imgur.com/"u003eu003c/au003e",
              contentPolicyHtml: "User contributions licensed under u003ca href="https://creativecommons.org/licenses/by-sa/3.0/"u003ecc by-sa 3.0 with attribution requiredu003c/au003e u003ca href="https://stackoverflow.com/legal/content-policy"u003e(content policy)u003c/au003e",
              allowUrls: true
              },
              noCode: true, onDemand: true,
              discardSelector: ".discard-answer"
              ,immediatelyShowMarkdownHelp:true
              });


              }
              });














              draft saved

              draft discarded


















              StackExchange.ready(
              function () {
              StackExchange.openid.initPostLogin('.new-post-login', 'https%3a%2f%2fmath.stackexchange.com%2fquestions%2f3002437%2fsum-of-reciprocal-numbers-of-combinations%23new-answer', 'question_page');
              }
              );

              Post as a guest















              Required, but never shown

























              2 Answers
              2






              active

              oldest

              votes








              2 Answers
              2






              active

              oldest

              votes









              active

              oldest

              votes






              active

              oldest

              votes








              up vote
              1
              down vote



              accepted










              That is known as the German tank problem, and is one of the fundamental Binomial Identities.






              share|cite|improve this answer

























                up vote
                1
                down vote



                accepted










                That is known as the German tank problem, and is one of the fundamental Binomial Identities.






                share|cite|improve this answer























                  up vote
                  1
                  down vote



                  accepted







                  up vote
                  1
                  down vote



                  accepted






                  That is known as the German tank problem, and is one of the fundamental Binomial Identities.






                  share|cite|improve this answer












                  That is known as the German tank problem, and is one of the fundamental Binomial Identities.







                  share|cite|improve this answer












                  share|cite|improve this answer



                  share|cite|improve this answer










                  answered Nov 17 at 15:54









                  G Cab

                  17.2k31237




                  17.2k31237






















                      up vote
                      0
                      down vote













                      This is very simple to prove. All you need to note is the following identity :
                      $$
                      frac 1{a(a+1)...(a+m)} = frac{1}{m}left( frac 1{a(a+1)...(a+m-1)} - frac 1{(a+1)...(a+m)}right)
                      $$



                      Now, consider the sum:
                      $$
                      sum_{n=k+1}^{k+m} frac 1{binom nk} = k! times sum_{i=1}^{i=m} frac{i!}{(k+i)!} \ = k! sum_{i=1}^{i=m}frac 1{(i+1)(i+2)...(i+k)} \ = k! sum_{i=1}^{i=m} frac{1}{k-1}left(frac{1}{(i+1)...(i+k-1)} - frac 1{(i+2)...(i+k)}right) \ = frac{k!}{k-1}sum_{i=1}^{i=m} left(frac{1}{(i+1)...(i+k-1)} - frac 1{(i+2)...(i+k)}right) \ = frac{k!}{k-1} left(frac 1{k!} - frac 1{(m+2)...(m+k)}right) \ = frac 1{k-1} - frac{k!}{(k-1)(m+2)...(m+k)}
                      $$



                      By letting $m$ go to infinity, the answer is clearly $frac 1{k-1}$, since the second term goes to zero.






                      share|cite|improve this answer

























                        up vote
                        0
                        down vote













                        This is very simple to prove. All you need to note is the following identity :
                        $$
                        frac 1{a(a+1)...(a+m)} = frac{1}{m}left( frac 1{a(a+1)...(a+m-1)} - frac 1{(a+1)...(a+m)}right)
                        $$



                        Now, consider the sum:
                        $$
                        sum_{n=k+1}^{k+m} frac 1{binom nk} = k! times sum_{i=1}^{i=m} frac{i!}{(k+i)!} \ = k! sum_{i=1}^{i=m}frac 1{(i+1)(i+2)...(i+k)} \ = k! sum_{i=1}^{i=m} frac{1}{k-1}left(frac{1}{(i+1)...(i+k-1)} - frac 1{(i+2)...(i+k)}right) \ = frac{k!}{k-1}sum_{i=1}^{i=m} left(frac{1}{(i+1)...(i+k-1)} - frac 1{(i+2)...(i+k)}right) \ = frac{k!}{k-1} left(frac 1{k!} - frac 1{(m+2)...(m+k)}right) \ = frac 1{k-1} - frac{k!}{(k-1)(m+2)...(m+k)}
                        $$



                        By letting $m$ go to infinity, the answer is clearly $frac 1{k-1}$, since the second term goes to zero.






                        share|cite|improve this answer























                          up vote
                          0
                          down vote










                          up vote
                          0
                          down vote









                          This is very simple to prove. All you need to note is the following identity :
                          $$
                          frac 1{a(a+1)...(a+m)} = frac{1}{m}left( frac 1{a(a+1)...(a+m-1)} - frac 1{(a+1)...(a+m)}right)
                          $$



                          Now, consider the sum:
                          $$
                          sum_{n=k+1}^{k+m} frac 1{binom nk} = k! times sum_{i=1}^{i=m} frac{i!}{(k+i)!} \ = k! sum_{i=1}^{i=m}frac 1{(i+1)(i+2)...(i+k)} \ = k! sum_{i=1}^{i=m} frac{1}{k-1}left(frac{1}{(i+1)...(i+k-1)} - frac 1{(i+2)...(i+k)}right) \ = frac{k!}{k-1}sum_{i=1}^{i=m} left(frac{1}{(i+1)...(i+k-1)} - frac 1{(i+2)...(i+k)}right) \ = frac{k!}{k-1} left(frac 1{k!} - frac 1{(m+2)...(m+k)}right) \ = frac 1{k-1} - frac{k!}{(k-1)(m+2)...(m+k)}
                          $$



                          By letting $m$ go to infinity, the answer is clearly $frac 1{k-1}$, since the second term goes to zero.






                          share|cite|improve this answer












                          This is very simple to prove. All you need to note is the following identity :
                          $$
                          frac 1{a(a+1)...(a+m)} = frac{1}{m}left( frac 1{a(a+1)...(a+m-1)} - frac 1{(a+1)...(a+m)}right)
                          $$



                          Now, consider the sum:
                          $$
                          sum_{n=k+1}^{k+m} frac 1{binom nk} = k! times sum_{i=1}^{i=m} frac{i!}{(k+i)!} \ = k! sum_{i=1}^{i=m}frac 1{(i+1)(i+2)...(i+k)} \ = k! sum_{i=1}^{i=m} frac{1}{k-1}left(frac{1}{(i+1)...(i+k-1)} - frac 1{(i+2)...(i+k)}right) \ = frac{k!}{k-1}sum_{i=1}^{i=m} left(frac{1}{(i+1)...(i+k-1)} - frac 1{(i+2)...(i+k)}right) \ = frac{k!}{k-1} left(frac 1{k!} - frac 1{(m+2)...(m+k)}right) \ = frac 1{k-1} - frac{k!}{(k-1)(m+2)...(m+k)}
                          $$



                          By letting $m$ go to infinity, the answer is clearly $frac 1{k-1}$, since the second term goes to zero.







                          share|cite|improve this answer












                          share|cite|improve this answer



                          share|cite|improve this answer










                          answered Nov 17 at 16:13









                          астон вілла олоф мэллбэрг

                          36.9k33376




                          36.9k33376






























                              draft saved

                              draft discarded




















































                              Thanks for contributing an answer to Mathematics Stack Exchange!


                              • Please be sure to answer the question. Provide details and share your research!

                              But avoid



                              • Asking for help, clarification, or responding to other answers.

                              • Making statements based on opinion; back them up with references or personal experience.


                              Use MathJax to format equations. MathJax reference.


                              To learn more, see our tips on writing great answers.





                              Some of your past answers have not been well-received, and you're in danger of being blocked from answering.


                              Please pay close attention to the following guidance:


                              • Please be sure to answer the question. Provide details and share your research!

                              But avoid



                              • Asking for help, clarification, or responding to other answers.

                              • Making statements based on opinion; back them up with references or personal experience.


                              To learn more, see our tips on writing great answers.




                              draft saved


                              draft discarded














                              StackExchange.ready(
                              function () {
                              StackExchange.openid.initPostLogin('.new-post-login', 'https%3a%2f%2fmath.stackexchange.com%2fquestions%2f3002437%2fsum-of-reciprocal-numbers-of-combinations%23new-answer', 'question_page');
                              }
                              );

                              Post as a guest















                              Required, but never shown





















































                              Required, but never shown














                              Required, but never shown












                              Required, but never shown







                              Required, but never shown

































                              Required, but never shown














                              Required, but never shown












                              Required, but never shown







                              Required, but never shown







                              Popular posts from this blog

                              Plaza Victoria

                              In PowerPoint, is there a keyboard shortcut for bulleted / numbered list?

                              How to put 3 figures in Latex with 2 figures side by side and 1 below these side by side images but in...